Proof Verification of Baby Rudin 3.3











up vote
0
down vote

favorite












I am currently studying Rudin's Foundations of Real Analysis. I wrote a solution to Rudin's problem 3.3, and wanted to get some input on my proof. I appreciate any suggestions on the technicalities, style, and clarity.



Problem:




If $s_1=sqrt2$, and
$s_{n+1} = sqrt{2+sqrt{s_n}}$ where $n = 1, 2, 3, ...$ prove that {${s_n}$} converges, and that $s_n<2$ for $n = 1, 2, 3, ...$




My proof:



We first show $s_n<2$ for $n = 1, 2, 3, ...$ by induction.
When $n = 1$, it is clear that $sqrt2 < 2$.



If $s_{n-1}$ < 2, then



$0<sqrt{2+sqrt{s_{n-1}}} < 2$



$leftrightarrow 2+sqrt{s_{n-1}}<4$



$leftrightarrow sqrt{s_{n-1}} < 2$



$leftrightarrow s_{n-1} < 4$



But $s_{n-1} < 2$ by induction hypothesis. Thus $s_n<2$ for $n = 1, 2, 3, ...$ as desired.



We next show that {$s_{n}$} is monotonically increasing by induction.
It is clear that $s_1leq s_2$.
We need to show $s_n leq s_{n+1}$ for all $n geq 2$. Assume $s_{n-1} leq s_{n}$. Then,



$s_n leq s_{n+1}$



$leftrightarrow 0 < sqrt{2+sqrt{s_{n-1}}} leq sqrt{2+sqrt{s_{n}}}$



$leftrightarrow 2+sqrt{s_{n-1}} leq 2+sqrt{s_{n}}$



$leftrightarrow sqrt{s_{n-1}} leq sqrt{s_{n}}$



$leftrightarrow s_{n-1} leq s_{n}$



But $s_{n-1} leq s_{n}$ by the induction hypothesis. Thus the sequence is monotonically increasing.



Since {$s_n$} is bounded above and monotonically increasing, it converges.










share|cite|improve this question




























    up vote
    0
    down vote

    favorite












    I am currently studying Rudin's Foundations of Real Analysis. I wrote a solution to Rudin's problem 3.3, and wanted to get some input on my proof. I appreciate any suggestions on the technicalities, style, and clarity.



    Problem:




    If $s_1=sqrt2$, and
    $s_{n+1} = sqrt{2+sqrt{s_n}}$ where $n = 1, 2, 3, ...$ prove that {${s_n}$} converges, and that $s_n<2$ for $n = 1, 2, 3, ...$




    My proof:



    We first show $s_n<2$ for $n = 1, 2, 3, ...$ by induction.
    When $n = 1$, it is clear that $sqrt2 < 2$.



    If $s_{n-1}$ < 2, then



    $0<sqrt{2+sqrt{s_{n-1}}} < 2$



    $leftrightarrow 2+sqrt{s_{n-1}}<4$



    $leftrightarrow sqrt{s_{n-1}} < 2$



    $leftrightarrow s_{n-1} < 4$



    But $s_{n-1} < 2$ by induction hypothesis. Thus $s_n<2$ for $n = 1, 2, 3, ...$ as desired.



    We next show that {$s_{n}$} is monotonically increasing by induction.
    It is clear that $s_1leq s_2$.
    We need to show $s_n leq s_{n+1}$ for all $n geq 2$. Assume $s_{n-1} leq s_{n}$. Then,



    $s_n leq s_{n+1}$



    $leftrightarrow 0 < sqrt{2+sqrt{s_{n-1}}} leq sqrt{2+sqrt{s_{n}}}$



    $leftrightarrow 2+sqrt{s_{n-1}} leq 2+sqrt{s_{n}}$



    $leftrightarrow sqrt{s_{n-1}} leq sqrt{s_{n}}$



    $leftrightarrow s_{n-1} leq s_{n}$



    But $s_{n-1} leq s_{n}$ by the induction hypothesis. Thus the sequence is monotonically increasing.



    Since {$s_n$} is bounded above and monotonically increasing, it converges.










    share|cite|improve this question


























      up vote
      0
      down vote

      favorite









      up vote
      0
      down vote

      favorite











      I am currently studying Rudin's Foundations of Real Analysis. I wrote a solution to Rudin's problem 3.3, and wanted to get some input on my proof. I appreciate any suggestions on the technicalities, style, and clarity.



      Problem:




      If $s_1=sqrt2$, and
      $s_{n+1} = sqrt{2+sqrt{s_n}}$ where $n = 1, 2, 3, ...$ prove that {${s_n}$} converges, and that $s_n<2$ for $n = 1, 2, 3, ...$




      My proof:



      We first show $s_n<2$ for $n = 1, 2, 3, ...$ by induction.
      When $n = 1$, it is clear that $sqrt2 < 2$.



      If $s_{n-1}$ < 2, then



      $0<sqrt{2+sqrt{s_{n-1}}} < 2$



      $leftrightarrow 2+sqrt{s_{n-1}}<4$



      $leftrightarrow sqrt{s_{n-1}} < 2$



      $leftrightarrow s_{n-1} < 4$



      But $s_{n-1} < 2$ by induction hypothesis. Thus $s_n<2$ for $n = 1, 2, 3, ...$ as desired.



      We next show that {$s_{n}$} is monotonically increasing by induction.
      It is clear that $s_1leq s_2$.
      We need to show $s_n leq s_{n+1}$ for all $n geq 2$. Assume $s_{n-1} leq s_{n}$. Then,



      $s_n leq s_{n+1}$



      $leftrightarrow 0 < sqrt{2+sqrt{s_{n-1}}} leq sqrt{2+sqrt{s_{n}}}$



      $leftrightarrow 2+sqrt{s_{n-1}} leq 2+sqrt{s_{n}}$



      $leftrightarrow sqrt{s_{n-1}} leq sqrt{s_{n}}$



      $leftrightarrow s_{n-1} leq s_{n}$



      But $s_{n-1} leq s_{n}$ by the induction hypothesis. Thus the sequence is monotonically increasing.



      Since {$s_n$} is bounded above and monotonically increasing, it converges.










      share|cite|improve this question















      I am currently studying Rudin's Foundations of Real Analysis. I wrote a solution to Rudin's problem 3.3, and wanted to get some input on my proof. I appreciate any suggestions on the technicalities, style, and clarity.



      Problem:




      If $s_1=sqrt2$, and
      $s_{n+1} = sqrt{2+sqrt{s_n}}$ where $n = 1, 2, 3, ...$ prove that {${s_n}$} converges, and that $s_n<2$ for $n = 1, 2, 3, ...$




      My proof:



      We first show $s_n<2$ for $n = 1, 2, 3, ...$ by induction.
      When $n = 1$, it is clear that $sqrt2 < 2$.



      If $s_{n-1}$ < 2, then



      $0<sqrt{2+sqrt{s_{n-1}}} < 2$



      $leftrightarrow 2+sqrt{s_{n-1}}<4$



      $leftrightarrow sqrt{s_{n-1}} < 2$



      $leftrightarrow s_{n-1} < 4$



      But $s_{n-1} < 2$ by induction hypothesis. Thus $s_n<2$ for $n = 1, 2, 3, ...$ as desired.



      We next show that {$s_{n}$} is monotonically increasing by induction.
      It is clear that $s_1leq s_2$.
      We need to show $s_n leq s_{n+1}$ for all $n geq 2$. Assume $s_{n-1} leq s_{n}$. Then,



      $s_n leq s_{n+1}$



      $leftrightarrow 0 < sqrt{2+sqrt{s_{n-1}}} leq sqrt{2+sqrt{s_{n}}}$



      $leftrightarrow 2+sqrt{s_{n-1}} leq 2+sqrt{s_{n}}$



      $leftrightarrow sqrt{s_{n-1}} leq sqrt{s_{n}}$



      $leftrightarrow s_{n-1} leq s_{n}$



      But $s_{n-1} leq s_{n}$ by the induction hypothesis. Thus the sequence is monotonically increasing.



      Since {$s_n$} is bounded above and monotonically increasing, it converges.







      sequences-and-series






      share|cite|improve this question















      share|cite|improve this question













      share|cite|improve this question




      share|cite|improve this question








      edited 18 hours ago

























      asked Nov 5 at 5:20









      b_chao

      143




      143






















          1 Answer
          1






          active

          oldest

          votes

















          up vote
          0
          down vote














          1. For the first proof, Also, I would show a series of forwards implications rather than the string if "if and only if" statements you are using, since your proof doesn't require both directions.


          In this spirit, your first inductive proof, upper-bounding $s_{n}$ by $2$, could be re-phrased as follows:



          We claim $s_{n} < 2$ for all $n$ by induction.



          For $n = 1$ this is clear. Assume $s_{n-1} < 2$. Then since the square root function is monotonically increasing on the non-negative reals,
          begin{eqnarray}
          sqrt{s_{n-1}} < 2 \
          2 + sqrt{s_{n-1}} < 4 \
          sqrt{2 + sqrt{s_{n-1}}} < 2
          end{eqnarray}

          Since $s_n = sqrt{2 + sqrt{s_{n-1}}}$ this completes the inductive step.



          I made a point of saying the square-root function is monotonic. This is fairly pedantic, and I don't think is really needed, but given that your argument implicitly uses this fact, it doesn't hurt.




          1. Your second proof has what I believe to be a typo.



          We need show $s_n leq s_{n−1}$ for all $n geq 2$. Assume $s_n leq s_{n-1}$.




          At first glance, I wasn't sure if this was a typo or if you meant to do a proof by contradiction. I think what you meant to show was $s_n leq s_{n+1}$. In that case, replacing $s_{n-1}$ with $s_{n+1}$ where appropriate should be fine, and stylistically I would say the same things as regarding your first proof.






          share|cite|improve this answer








          New contributor




          Akhil Jalan is a new contributor to this site. Take care in asking for clarification, commenting, and answering.
          Check out our Code of Conduct.


















          • Thanks for the review! I corrected the typos you mentioned.
            – b_chao
            18 hours ago











          Your Answer





          StackExchange.ifUsing("editor", function () {
          return StackExchange.using("mathjaxEditing", function () {
          StackExchange.MarkdownEditor.creationCallbacks.add(function (editor, postfix) {
          StackExchange.mathjaxEditing.prepareWmdForMathJax(editor, postfix, [["$", "$"], ["\\(","\\)"]]);
          });
          });
          }, "mathjax-editing");

          StackExchange.ready(function() {
          var channelOptions = {
          tags: "".split(" "),
          id: "69"
          };
          initTagRenderer("".split(" "), "".split(" "), channelOptions);

          StackExchange.using("externalEditor", function() {
          // Have to fire editor after snippets, if snippets enabled
          if (StackExchange.settings.snippets.snippetsEnabled) {
          StackExchange.using("snippets", function() {
          createEditor();
          });
          }
          else {
          createEditor();
          }
          });

          function createEditor() {
          StackExchange.prepareEditor({
          heartbeatType: 'answer',
          convertImagesToLinks: true,
          noModals: true,
          showLowRepImageUploadWarning: true,
          reputationToPostImages: 10,
          bindNavPrevention: true,
          postfix: "",
          imageUploader: {
          brandingHtml: "Powered by u003ca class="icon-imgur-white" href="https://imgur.com/"u003eu003c/au003e",
          contentPolicyHtml: "User contributions licensed under u003ca href="https://creativecommons.org/licenses/by-sa/3.0/"u003ecc by-sa 3.0 with attribution requiredu003c/au003e u003ca href="https://stackoverflow.com/legal/content-policy"u003e(content policy)u003c/au003e",
          allowUrls: true
          },
          noCode: true, onDemand: true,
          discardSelector: ".discard-answer"
          ,immediatelyShowMarkdownHelp:true
          });


          }
          });














           

          draft saved


          draft discarded


















          StackExchange.ready(
          function () {
          StackExchange.openid.initPostLogin('.new-post-login', 'https%3a%2f%2fmath.stackexchange.com%2fquestions%2f2985303%2fproof-verification-of-baby-rudin-3-3%23new-answer', 'question_page');
          }
          );

          Post as a guest















          Required, but never shown

























          1 Answer
          1






          active

          oldest

          votes








          1 Answer
          1






          active

          oldest

          votes









          active

          oldest

          votes






          active

          oldest

          votes








          up vote
          0
          down vote














          1. For the first proof, Also, I would show a series of forwards implications rather than the string if "if and only if" statements you are using, since your proof doesn't require both directions.


          In this spirit, your first inductive proof, upper-bounding $s_{n}$ by $2$, could be re-phrased as follows:



          We claim $s_{n} < 2$ for all $n$ by induction.



          For $n = 1$ this is clear. Assume $s_{n-1} < 2$. Then since the square root function is monotonically increasing on the non-negative reals,
          begin{eqnarray}
          sqrt{s_{n-1}} < 2 \
          2 + sqrt{s_{n-1}} < 4 \
          sqrt{2 + sqrt{s_{n-1}}} < 2
          end{eqnarray}

          Since $s_n = sqrt{2 + sqrt{s_{n-1}}}$ this completes the inductive step.



          I made a point of saying the square-root function is monotonic. This is fairly pedantic, and I don't think is really needed, but given that your argument implicitly uses this fact, it doesn't hurt.




          1. Your second proof has what I believe to be a typo.



          We need show $s_n leq s_{n−1}$ for all $n geq 2$. Assume $s_n leq s_{n-1}$.




          At first glance, I wasn't sure if this was a typo or if you meant to do a proof by contradiction. I think what you meant to show was $s_n leq s_{n+1}$. In that case, replacing $s_{n-1}$ with $s_{n+1}$ where appropriate should be fine, and stylistically I would say the same things as regarding your first proof.






          share|cite|improve this answer








          New contributor




          Akhil Jalan is a new contributor to this site. Take care in asking for clarification, commenting, and answering.
          Check out our Code of Conduct.


















          • Thanks for the review! I corrected the typos you mentioned.
            – b_chao
            18 hours ago















          up vote
          0
          down vote














          1. For the first proof, Also, I would show a series of forwards implications rather than the string if "if and only if" statements you are using, since your proof doesn't require both directions.


          In this spirit, your first inductive proof, upper-bounding $s_{n}$ by $2$, could be re-phrased as follows:



          We claim $s_{n} < 2$ for all $n$ by induction.



          For $n = 1$ this is clear. Assume $s_{n-1} < 2$. Then since the square root function is monotonically increasing on the non-negative reals,
          begin{eqnarray}
          sqrt{s_{n-1}} < 2 \
          2 + sqrt{s_{n-1}} < 4 \
          sqrt{2 + sqrt{s_{n-1}}} < 2
          end{eqnarray}

          Since $s_n = sqrt{2 + sqrt{s_{n-1}}}$ this completes the inductive step.



          I made a point of saying the square-root function is monotonic. This is fairly pedantic, and I don't think is really needed, but given that your argument implicitly uses this fact, it doesn't hurt.




          1. Your second proof has what I believe to be a typo.



          We need show $s_n leq s_{n−1}$ for all $n geq 2$. Assume $s_n leq s_{n-1}$.




          At first glance, I wasn't sure if this was a typo or if you meant to do a proof by contradiction. I think what you meant to show was $s_n leq s_{n+1}$. In that case, replacing $s_{n-1}$ with $s_{n+1}$ where appropriate should be fine, and stylistically I would say the same things as regarding your first proof.






          share|cite|improve this answer








          New contributor




          Akhil Jalan is a new contributor to this site. Take care in asking for clarification, commenting, and answering.
          Check out our Code of Conduct.


















          • Thanks for the review! I corrected the typos you mentioned.
            – b_chao
            18 hours ago













          up vote
          0
          down vote










          up vote
          0
          down vote










          1. For the first proof, Also, I would show a series of forwards implications rather than the string if "if and only if" statements you are using, since your proof doesn't require both directions.


          In this spirit, your first inductive proof, upper-bounding $s_{n}$ by $2$, could be re-phrased as follows:



          We claim $s_{n} < 2$ for all $n$ by induction.



          For $n = 1$ this is clear. Assume $s_{n-1} < 2$. Then since the square root function is monotonically increasing on the non-negative reals,
          begin{eqnarray}
          sqrt{s_{n-1}} < 2 \
          2 + sqrt{s_{n-1}} < 4 \
          sqrt{2 + sqrt{s_{n-1}}} < 2
          end{eqnarray}

          Since $s_n = sqrt{2 + sqrt{s_{n-1}}}$ this completes the inductive step.



          I made a point of saying the square-root function is monotonic. This is fairly pedantic, and I don't think is really needed, but given that your argument implicitly uses this fact, it doesn't hurt.




          1. Your second proof has what I believe to be a typo.



          We need show $s_n leq s_{n−1}$ for all $n geq 2$. Assume $s_n leq s_{n-1}$.




          At first glance, I wasn't sure if this was a typo or if you meant to do a proof by contradiction. I think what you meant to show was $s_n leq s_{n+1}$. In that case, replacing $s_{n-1}$ with $s_{n+1}$ where appropriate should be fine, and stylistically I would say the same things as regarding your first proof.






          share|cite|improve this answer








          New contributor




          Akhil Jalan is a new contributor to this site. Take care in asking for clarification, commenting, and answering.
          Check out our Code of Conduct.










          1. For the first proof, Also, I would show a series of forwards implications rather than the string if "if and only if" statements you are using, since your proof doesn't require both directions.


          In this spirit, your first inductive proof, upper-bounding $s_{n}$ by $2$, could be re-phrased as follows:



          We claim $s_{n} < 2$ for all $n$ by induction.



          For $n = 1$ this is clear. Assume $s_{n-1} < 2$. Then since the square root function is monotonically increasing on the non-negative reals,
          begin{eqnarray}
          sqrt{s_{n-1}} < 2 \
          2 + sqrt{s_{n-1}} < 4 \
          sqrt{2 + sqrt{s_{n-1}}} < 2
          end{eqnarray}

          Since $s_n = sqrt{2 + sqrt{s_{n-1}}}$ this completes the inductive step.



          I made a point of saying the square-root function is monotonic. This is fairly pedantic, and I don't think is really needed, but given that your argument implicitly uses this fact, it doesn't hurt.




          1. Your second proof has what I believe to be a typo.



          We need show $s_n leq s_{n−1}$ for all $n geq 2$. Assume $s_n leq s_{n-1}$.




          At first glance, I wasn't sure if this was a typo or if you meant to do a proof by contradiction. I think what you meant to show was $s_n leq s_{n+1}$. In that case, replacing $s_{n-1}$ with $s_{n+1}$ where appropriate should be fine, and stylistically I would say the same things as regarding your first proof.







          share|cite|improve this answer








          New contributor




          Akhil Jalan is a new contributor to this site. Take care in asking for clarification, commenting, and answering.
          Check out our Code of Conduct.









          share|cite|improve this answer



          share|cite|improve this answer






          New contributor




          Akhil Jalan is a new contributor to this site. Take care in asking for clarification, commenting, and answering.
          Check out our Code of Conduct.









          answered yesterday









          Akhil Jalan

          465




          465




          New contributor




          Akhil Jalan is a new contributor to this site. Take care in asking for clarification, commenting, and answering.
          Check out our Code of Conduct.





          New contributor





          Akhil Jalan is a new contributor to this site. Take care in asking for clarification, commenting, and answering.
          Check out our Code of Conduct.






          Akhil Jalan is a new contributor to this site. Take care in asking for clarification, commenting, and answering.
          Check out our Code of Conduct.












          • Thanks for the review! I corrected the typos you mentioned.
            – b_chao
            18 hours ago


















          • Thanks for the review! I corrected the typos you mentioned.
            – b_chao
            18 hours ago
















          Thanks for the review! I corrected the typos you mentioned.
          – b_chao
          18 hours ago




          Thanks for the review! I corrected the typos you mentioned.
          – b_chao
          18 hours ago


















           

          draft saved


          draft discarded



















































           


          draft saved


          draft discarded














          StackExchange.ready(
          function () {
          StackExchange.openid.initPostLogin('.new-post-login', 'https%3a%2f%2fmath.stackexchange.com%2fquestions%2f2985303%2fproof-verification-of-baby-rudin-3-3%23new-answer', 'question_page');
          }
          );

          Post as a guest















          Required, but never shown





















































          Required, but never shown














          Required, but never shown












          Required, but never shown







          Required, but never shown

































          Required, but never shown














          Required, but never shown












          Required, but never shown







          Required, but never shown







          Popular posts from this blog

          Can a sorcerer learn a 5th-level spell early by creating spell slots using the Font of Magic feature?

          Does disintegrating a polymorphed enemy still kill it after the 2018 errata?

          A Topological Invariant for $pi_3(U(n))$